Good DS question - Cogeo II

This topic has expert replies
Master | Next Rank: 500 Posts
Posts: 110
Joined: Thu Oct 23, 2008 9:12 am
Location: India
Thanked: 9 times
Followed by:1 members

Good DS question - Cogeo II

by austin » Wed Apr 08, 2009 11:19 pm
What is the slope of line p?
1. The x-intercept is twice the y intercept
2. The line passes through I and III quadrants

OA in a bit

Legendary Member
Posts: 594
Joined: Thu Aug 14, 2008 11:51 pm
Thanked: 12 times

by nervesofsteel » Thu Apr 09, 2009 12:35 am
C for me...

A) insufficient as if line passes through (0,1) and ( -2,0) slope is 1/2
id it passe through (0,1) and (2,0) the slope becomes -1/2.. so insufficient..

B) Line passing through quads cannot give an idea about slope...

if line passes through I and III quad.. and intercept on X axis is double the intercept on Y axis..then slope must be 1/2...

Master | Next Rank: 500 Posts
Posts: 145
Joined: Mon Feb 16, 2009 8:41 am
Thanked: 2 times
Followed by:2 members

by anshulseth » Fri Apr 10, 2009 1:07 am
Lets use a generic eq of a line:
y=mx+c

If X-intercept = 2*Y-intercept

X-Intercept = -c/m
Y-Intercept = c

So, as per Stmt I: -c/m=2c
or m =-1/2

Sufficient.

Stmt II: we just know line passes thro center.
No more info.
Insufficient

Thus, A is the answer.
Asset

Legendary Member
Posts: 876
Joined: Thu Apr 10, 2008 8:14 am
Thanked: 13 times

by ketkoag » Wed Apr 15, 2009 3:31 am
C for me as well.
by statement 1 slope can be negative or positive. hence insufficient.
by (2) we cannot find the actual value. hence insufficient.
Combining both, we get the slope as 1/2. hence sufficient.

Master | Next Rank: 500 Posts
Posts: 418
Joined: Wed Jun 11, 2008 5:29 am
Thanked: 65 times

by bluementor » Wed Apr 15, 2009 3:51 am
One more vote for A, same reasoning as anshulseth....
nervesofsteel wrote: A) insufficient as if line passes through (0,1) and ( -2,0) slope is 1/2
I don't think this satisfies the condition in statement 1.

Statement 1 simply says:

c = 2*(-c/m), which is sufficient to answer for m. Hence A.

-BM-

User avatar
GMAT Instructor
Posts: 3650
Joined: Wed Jan 21, 2009 4:27 am
Location: India
Thanked: 267 times
Followed by:80 members
GMAT Score:760

by sanju09 » Sat Apr 18, 2009 5:27 am
Slope of a line = - y-intercept/x-intercept.

For (1) x-intercept = 2 × y intercept, slope can be answered.

For (2) if the line passes through I and III quadrants along with passing through II or IV quadrant as well, or along with passing through the origin (no intercepts are possible in this case), the slope must be positive; but what exactly will be the value? No answer.

Take A from me.
The mind is everything. What you think you become. -Lord Buddha



Sanjeev K Saxena
Quantitative Instructor
The Princeton Review - Manya Abroad
Lucknow-226001

www.manyagroup.com

User avatar
Master | Next Rank: 500 Posts
Posts: 246
Joined: Mon May 19, 2008 7:34 am
Location: Texaco Gas Station
Thanked: 7 times

by cubicle_bound_misfit » Mon Apr 20, 2009 8:50 pm
Just one question to all the gurus.
Is there any way to find out that if a straight line goes thru I and III quad, and if x-intercept is twice y-intercept then, how many solutions possible?
Cubicle Bound Misfit

Master | Next Rank: 500 Posts
Posts: 110
Joined: Thu Oct 23, 2008 9:12 am
Location: India
Thanked: 9 times
Followed by:1 members

by austin » Mon Apr 20, 2009 8:58 pm
If a line passes through the first and third quadrants, the slope of the line could be +ve or -ve. The correct answer to this question is C.

NOTE: For a line passing thru 1 and 3 quad, slope is +ve
For a line passing thru 2 and 4 quad, slope is -ve

Master | Next Rank: 500 Posts
Posts: 418
Joined: Wed Jun 11, 2008 5:29 am
Thanked: 65 times

by bluementor » Tue Apr 21, 2009 2:08 am
I don't quite understand how C can be the answer.

We know that:

slope = - (y-intercept)/(x-intercept)

Statement 1 tells us that:

x-intercept = 2 * (y-intercept) ,

which is enough to answer the question. This also certainly proves that the slope can only be -1/2. So why not A?

-BM-
Last edited by bluementor on Tue Apr 21, 2009 4:41 am, edited 1 time in total.

User avatar
Master | Next Rank: 500 Posts
Posts: 229
Joined: Tue Jan 13, 2009 6:56 am
Thanked: 8 times
GMAT Score:700

by Uri » Tue Apr 21, 2009 2:17 am
why do we need (2) for the statement to be true? (1) alone provides us with the solution So, answer should be (A).
nervesofsteel wrote:C for me...

A) insufficient as if line passes through (0,1) and ( -2,0) slope is 1/2
id it passe through (0,1) and (2,0) the slope becomes -1/2.. so insufficient..
for (0,1) and (-2,0) the x-intercept is -2 times the y-intercept. if intercept means absolute value, then this would be correct. otherwise, (1) is sufficient. so, can any expert please help us with the answer?


austin, , can you please tell us the source of this question?
Last edited by Uri on Mon Apr 27, 2009 3:16 am, edited 1 time in total.

Master | Next Rank: 500 Posts
Posts: 110
Joined: Thu Oct 23, 2008 9:12 am
Location: India
Thanked: 9 times
Followed by:1 members

by austin » Tue Apr 21, 2009 2:57 am
When we say intercept of three, the distance from (0,0) is 3...but, is it in the positive or negative direction???

If a line passes through (3,0) or (-3,0), the intercept is still 3.. but the slopes could be different....

TRY YOURSELF: You can draw two lines: one through (3,0) and the other through (-3,0) with x-intercept twice the y intercept... The slope is 1/2 in one case and -1/2 the other case....

NOTE: If the angle made by the line with the +ve direction of the x-axis is acute (<90), then slope = tan (angle) = +ve (tan is +ve in first quadrant)

If the angle made by the line with the +ve direction of the x-axis is obtuse (>90), then slope = tan (angle) = -ve (tan is -ve in second quadrant)

SO WE NEED TO THROUGH WHICH QUADRANTs THE LINE PASSES THROUGH.

Master | Next Rank: 500 Posts
Posts: 110
Joined: Thu Oct 23, 2008 9:12 am
Location: India
Thanked: 9 times
Followed by:1 members

by austin » Tue Apr 21, 2009 3:19 am
I am quoting fleurdelisse's question and the discussion link here:

In the rectangular coordinate, does line k interest quandrant II? (quandrant II being the quandrant where x<0 and y>0)
(1) slope of k is -1/6
(2) y-intercept of k is -6

https://www.beatthegmat.com/line-and-slope-t35149.html

This, I think will help you understand the concept of intercept....

Observe statement 2: y intercept is -6. This means the line cuts at (0,-6).

Master | Next Rank: 500 Posts
Posts: 418
Joined: Wed Jun 11, 2008 5:29 am
Thanked: 65 times

by bluementor » Tue Apr 21, 2009 3:58 am
austin, thanks for responding.

Pls correct me if i am wrong, but the way i see it your explanation is contradictory.

In this post you mentioned:
austin wrote:When we say intercept of three, the distance from (0,0) is 3...but, is it in the positive or negative direction???

If a line passes through (3,0) or (-3,0), the intercept is still 3.. but the slopes could be different....
Here you are basically treating the intercept value as an absolute. If the y-intercept of a line is 3, then it could be intersecting the y-axis at (0,3) or (0,-3).

However, in the following post:
austin wrote:Observe statement 2: y intercept is -6. This means the line cuts at (0,-6).
Here you acknowledge that the sign of the y-intercept corresponds directly to a single point on the y-axis.

So which is it? I believe the later is true. If the y-intercept is 3, then the line can ONLY intersect (0,3), and NOT (0,-3). This is the reason why I feel statement 1 alone is sufficient to answer the original question in this post.

-BM-

Master | Next Rank: 500 Posts
Posts: 110
Joined: Thu Oct 23, 2008 9:12 am
Location: India
Thanked: 9 times
Followed by:1 members

by austin » Tue Apr 21, 2009 4:18 am
When we say intercept of 3, it means distance is 3...

We either say intercept of 3 in the negative direction of x-axis or say intercept is -3....both mean it passes thru (-3,0)

I used the latter example (fleurdelisse's problem) to convey that when you use the intercept to determine the slope you need to be careful....

"Making an intercept of 3" is different from "making an intercept of 3 with the positive direction of x-axis".. the latter is much more explicit.. the former is a bit tricky... the line can pass thru (3,0) or (-3,0) leading to different slopes and still we say intercept is 3....

GMAT/MBA Expert

User avatar
GMAT Instructor
Posts: 2621
Joined: Mon Jun 02, 2008 3:17 am
Location: Montreal
Thanked: 1090 times
Followed by:355 members
GMAT Score:780

Re: Good DS question - Cogeo II

by Ian Stewart » Tue Apr 21, 2009 4:21 am
austin wrote:What is the slope of line p?
1. The x-intercept is twice the y intercept
2. The line passes through I and III quadrants

OA in a bit
I'm also curious about the source. I'm going with E here, on a technicality. If the x-intercept and y-intercept are both zero, then the x-intercept is certainly two times the y-intercept, and as long as the slope is positive, the second statement will be true. So using both statements, the slope could be any positive number.

That's the only way to get the statements to agree, in fact; if the intercepts of line p are non-zero, then Statement 1 and Statement 2 are contradictory (Statement 1 guarantees the slope is negative, while Statement 2 guarantees the slope is positive).

We don't need any specialized formulas to analyze Statement 1, incidentally. Say the y-intercept of the line is (0,b), where b is nonzero. Then the x-intercept is (2b, 0), according to Statement 1. So the slope is:

(b - 0)/(0 - 2b) = -1/2

That will be true as long as b is not zero.
For online GMAT math tutoring, or to buy my higher-level Quant books and problem sets, contact me at ianstewartgmat at gmail.com

ianstewartgmat.com